Difference between revisions of "2016 AMC 10B Problems/Problem 7"

(Created page with "==Problem== The ratio of the measures of two acute angles is <math>5:4</math>, and the complement of one of these two angles is twice as large as the complement of the other....")
 
(Problem)
Line 4: Line 4:
  
 
<math>\textbf{(A)}\ 75\qquad\textbf{(B)}\ 90\qquad\textbf{(C)}\ 135\qquad\textbf{(D)}\ 150\qquad\textbf{(E)}\ 270</math>
 
<math>\textbf{(A)}\ 75\qquad\textbf{(B)}\ 90\qquad\textbf{(C)}\ 135\qquad\textbf{(D)}\ 150\qquad\textbf{(E)}\ 270</math>
 +
 +
 +
==Solution==
 +
 +
Set up a system of equations where <math>x</math> and <math>y</math> are the two acute angles with the assumption that <math>x</math> <math><</math> <math>y</math>:
 +
<math>5x</math> <math>=</math> <math>4y</math>

Revision as of 12:37, 21 February 2016

Problem

The ratio of the measures of two acute angles is $5:4$, and the complement of one of these two angles is twice as large as the complement of the other. What is the sum of the degree measures of the two angles?

$\textbf{(A)}\ 75\qquad\textbf{(B)}\ 90\qquad\textbf{(C)}\ 135\qquad\textbf{(D)}\ 150\qquad\textbf{(E)}\ 270$


Solution

Set up a system of equations where $x$ and $y$ are the two acute angles with the assumption that $x$ $<$ $y$: $5x$ $=$ $4y$